100% satisfaction guarantee Immediately available after payment Both online and in PDF No strings attached 4.2 TrustPilot
logo-home
Exam (elaborations)

ACLS COMPLETE latest Top Exam Questions and answers, 100% Accurate. Approved.

Rating
-
Sold
-
Pages
40
Grade
A+
Uploaded on
16-02-2023
Written in
2022/2023

ACLS COMPLETE latest Top Exam Questions and answers, 100% Accurate. Approved. __________ can help indicate coronary perfusion pressure - -Capnography ___________ correlates w/ ROSC - -High quality CPR 2nd degree mobitz - - 2nd degree wenckebach - - 62 yr old man suddenly expereinced difficulty speaking and left-sided weakness. He meets initial criteria for fibrinolytic therapy, and a CT scan of the brain is ordered. Which best describes the guidelines for antiplatelet and fibrinolytic therapy? (a) Give aspirin 160 to 325 mg to be chewed immediately (b) Give aspirin 160 mg and clopidogrel 75 mg orally (c) Give heparin if the CT scan is negative for hemorrhage (d) Hold aspirin for at least 24 hrs if rtPA is administered - -(d) Hold aspirin for at least 24 hrs if rtPA is administered A 35 year old woman has palpitations, light headedness, and a stable tachycardia. The monitor shows a regular narrow-complex QRS at a rate of 180/min. Vagal maneuvers have not been effective in terminating the rhythm. An IV has been established. What drug should be administered IV? - -Adenosine 6 mg A 35 year old woman has palpitations, light headedness, and a stable tachycardia. The monitor shows a regular narrow-complex QRS at a rate of 180/min. Vagal maneuvers have not been effective in terminating the rhythm. An IV has been established. What drug should be administered? - -adenosine 6 mg A 35 yr old female has palpitation, light-headedness, and a stable tachycardia. The monitor shows a regular narrow-complex QRS at a rate of 180/min. Vagal manuevers have not been effective in terminating the rhythm. An IV has been established. Which drug should be administered? (a) Adenosine 6 mg (b) Atropine 0.5 mg (c) Epinephrine 2 to 10 mcg/kg per minute (d) Lidocaine - -(a) Adenosine 6 mg A 35 yr old woman presents w/ a chief complaint of palpitations. She has no chest discomfort, shortness of breath, or light-headedness. Her BP is 120/78. On EKG, it shows she is in SVT. Which intervention is indicated first? (a) Adenosine 3 mg IV bolus (b) Adenosine 12 mg IV slow push (over 1 to 2 min) (c) Metoprolol 5 mg IV and repeat if necessary (d) Vagal manuevers - -(d) Vagal manuevers A 35-year-old woman has palpitations, light-headedness, and a stable tachycardia. The monitor shows a regular narrow-complex QRS at a rate of 180/min. Vagal maneuvers have not been effective in terminating the rhythm. An IV has been established. Which drug should be administered? - -Adenosine 6mg A 35-year-old woman presents with a chief complaint of palpitations. She has no chest discomfort, shortness of breath, or light-headedness. Her blood pressure is 120/78mm Hg. Which intervention is indicated first? - -Vagal maneuvers a 45 year old woman has a history of palpitations develops lightheadedness and palpitations. She has received adenosine 6 mg IV for SVT without conversion. BP 128/70. next step? - -adenosine 12 mg A 45 yr old woman with a hx of palpitations develops light-headedness and palpitations. She has received adenosine 6 mg IV for the rhythm shown here, without conversion of the rhythm. She is now extremely apprehensive. Her BP is 128/70 mm Hg. What is the next appropriate intervention? (a) Administer adenosine 12 mg IV (b) Perform unsynchronized cardioversion (c) Perform vagal maneuvers (d) Perform synchronized cardioversion - -(a) Administer adenosine 12 mg IV A 45-year-old woman with a history of palpitations develops light-headedness and palpitations. She has received adensoine 6mg IV for the rhythm shown here (SVT), without conversion of the rhythm. She is now extremely apprehensive. Her blood pressure si 128/70mm Hg. What is the next appropriate intervention? - -Administer adenosine 12 mg IV A 49-year-old man has retrosternal chest pain radiating into the left arm. The patient is diaphoretic, with associated shortness of breath. The blood pressure is 130/88 mm Hg, the heart rate is 110/min, the respiratory rate is 22 breaths/min, and the pulse oximetry value is 95%. The patient's 12-lead ECG shows ST-segment elevation in the anterior leads. First responders administered 160 mg of aspirin, and there is a patent peripheral IV. The pain is described as an 8 on a scale of 1 to 10 and is unrelieved after 3 doses of nitroglycerin. What is the next action? - -Administer 2 to 4 mg of morphine by slow IV bolus. A 49-year-old woman arrives in the emergency department with persistent epigastric pain. She had been taking oral antacids for the past 6 hours because she thought she had heartburn. The initial blood pressure is 118/72 mm Hg, the heart rate is 92/min and regular, the nonlabored respiratory rate is 14 breaths/min, and the pulse oximetry reading is 96%. Which is the most appropriate intervention to perform next? - -Obtain a 12 lead ECG. A 53-year-old man has shortness of breath, chest discomfort, and weakness. The patient's blood pressure is 102/59 mm Hg, the heart rate is 230/min, the respiratory rate is 16 breaths/min, and the pulse oximetry reading is 96%. The lead II ECG is displayed below. A patent peripheral IV is in place. What is the next action? - -Vagal maneuvers A 56-year-old man reports that he has palpitations but not chest pain or difficulty breathing. The blood pressure is 132/68 mm Hg, the pulse is 130/min and regular, the respiratory rate is 12 breaths/min, and the pulse oximetry reading is 95%. The lead II ECG displays a wide-complex tachycardia. What is the next action after establishing an IV and obtaining a 12-lead ECG? - -Seeking expert consultation A 57 year old woman has palpitations, chest discomfort, and tachycardia. The monitor shows a regular wide-complex QRS at a rate of 180 bpm. She becomes diaphoretic and her blood pressure is 80/60 mm Hg. What is the next action? - -cardioversion A 57 year old woman has palpitations, chest discomfort, and tachycardia. The monitor shows a regular wide-complex QRS at a rate of 180 bpm. She becomes diaphoretic and her blood pressure is 80/60 mm Hg. What is the next action? - -Perform immediate electrical cardioversion A 57-year-old woman has palpitation, chest discomfort, and tachycardia. The monitor shows a regular wide-QRS at a rate of 180/min. She becomes diaphoretic, and her blood pressure is 80/60 mm HG/ Which action do you take next? - -Perform electrical cordioversion A 62 year old man suddenly experienced difficulty speaking and left-sided weakness. He was brought into the emergency department. He meets initial criteria for fibrinolytic therapy, and a CT scan of the brain is ordered. What are the guidelines for antiplatelet and fibrinolytic therapy? - -Do not give ASA for at least 24 hours if rtPA is administered A 62 year old man suddenly experienced difficulty speaking and left-sided weakness. He was brought into the emergency department. He meets initial criteria for fibrinolytic therapy, and a CT scan of the brain is ordered. What are the guidelines for antiplatelet and fibrinolytic therapy? - -hold aspirin for at least 24 hours if rtPA is administered A 62-year-old man in the emergency department says that his heart is beating fast. He says he has no chest pain or shortness of breath. The blood pressure is 142/98 mm Hg, the pulse is 200/min, the respiratory rate is 14 breaths/min, and pulse oximetry is 95% on room air. What intervention should you perform next? - -Obtain a 12 lead ECG. A 62-year-old man suddenly experienced difficulty speaking and left-sided weakness. He meets initial criteria for fibrinolytic therapy, and a CT scan of the brain si ordered. Which best describes the guidelines for antiplatelet and fibrinolytic therapy? - -Hold aspirin for at least 24 hours if rtPA is administered A 67 yr old woman has palpitations, chest discomfort, and tachycardia. The monitor shows a regular wide-complex QRS at a rate of 180/min. She becomes diaphoretic, and her blood pressure is 80/60. Which action do you take next? (a) Establish IV access (b) Obtain a 12 lead EKG (c) Perform electrical cardioversion (d) Seek expert consultation - -(c) Perform electrical cardioversion A 68-year-old woman experienced a sudden onset of right arm weakness. EMS personnel measure a blood pressure of 140/90 mm Hg, a heart rate of 78/min, a nonlabored respiratory rate of 14 breaths/min, and a pulse oximetry reading of 97%. The lead II ECG displays sinus rhythm. What is the most appropriate action for the EMS team to perform next? - -Cincinnati Prehospital Stroke Scale assessment A monitored patient in the ICU developed a sudden onset of narrow-complex tachycardia at a rate of 220/min. The patient's blood pressure is 128/58 mm Hg, the PETCO2 is 38 mm Hg, and the pulse oximetry reading is 98%. There is vascular access at the left internal jugular vein, and the patient has not been given any vasoactive drugs. A 12-lead ECG confirms a supraventricular tachycardia with no evidence of ischemia or infarction. The heart rate has not responded to vagal maneuvers. What is the next recommended intervention? - -Adenosine 6mg IV push A monitored patient in the ICU developed a sudden onset of narrow-complex tachycardia at a rate of 220/min. The patient's blood pressure is 128/58 mm Hg, the PETCO2 is 38mm Hg, and the pulse oximetry reading is 98%. There is vascular access in the left arm, and the patient has not been given any vasoactive drugs. A 12-lead ECG confirm a supraventricular tachycardia with no evidence of ischemia or infarction. The heart rate has not responded to vagal maneuvers. what is your next action? - -Administer adenosine 6mg IV push A monitored pt in the ICU developed a sudden onset of narrow-complex tachycardia at a rate of 200. Pt's BP is 128/58, PETCO2 is 38, and pulse oximetry reading is 98%. There is vascular access in the left arm, and pt has not been given any vasoactive drugs. 12 lead EKG confirms a supraventricular tachycardia w/ no evidence of ischemia or infarction. Heart rate has not responded to vagal manuevers. What is your next action? (a) Administer adenosine 6 mg IV push (b) Administer amiodarone 300 mg IV push (c) Perform synchronized cardioversion at 50 J (d) Perform synchronized cardioversion at 200 J - -(a) Administer adenosine 6 mg IV push A patient becomes unresponsive. You are uncertain if a faint pulse is present with the rhythm below. What is your next action? - -CPR A patient becomes unresponsive. You are uncertain if a faint pulse is present. An IV is in place. Which action do you take next? (a) Begin transcutaneous pacing (b) Start high-quality CPR (c) Administer atropine 1 mg (d) Administer epinephrine 1 mg IV - -(b) Start high-quality CPR A patient becomes unresponsive. You are uncertain if a faint pulse is present. They rhythm shown here is seen on the cardiac monitor. An IV is in pace. Which action do you take next? - -Start high-quality CPR A patient has a rapid irregular wide-complex tachycardia. The ventricular rate is 138 bpm. He is asymptomatic, with a blood pressure of 110/70. He has a history of angina. What action is recommended next? (a) Giving adenosine 6 mg IV bolus (b) Giving lidocaine 1 to 1.5 mg IV bolus (c) Performing synchroniczed cardioversion (d) Seeking expert consultation - -(d) Seeking expert consultation A patient has a rapid irregular wide-complex tachycardia. The ventricular rate is 138/min. He is asymptomatic, with a blood pressure of 110/70 mm Hg. He has a history of angina. What action is recommended next? - -Seeking expert consultation A patient has a rapid irregular wide-complex tachycardia. The ventricular rate is 138/min. He is asymptomatic, with a blood pressure of 110/70 mm Hg. He has a history of angina. What should you do? - -seek expert consultation A patient has a rapid irregular wide-complex tachycardia. The ventricular rate is 138/min. He is asymptomatic, with a blood pressure of 110/70 mm Hg. He has a history of angina. What should you do? - -Seek expert consultation A patient has a witnessed loss of consciousness. The lead II ECG reveals this rhythm: What is the appropriate next intervention? - -Defibrillation A patient has been resuscitated from cardiac arrest. During post-ROSC treatment, the patient becomes unresponsive, with the rhythm shown here. Which action is indicated next? - -Give an immediate unsynchronized high-energy shock (defibrillation dose) a patient has been resuscitated from cardiac arrest. During post-ROSC treatment, the patient becomes unresponsive, with the rhythm VT. What tx is indicated - -unsynchronized high energy shock A patient has been rususcitated from cardiac arrest. During post-ROSC treatment, pt becomes unresponsive, with ventricular fibrillation. Which action is indicated next? (a) Give an immediate unsynchronized high-energy shock (defibrillation dose) (b) Give lidocaine 1 to 1.6 mg/kg IV (c) Perform synchronized cardioversion (d) Repeat amiodarone 300 mg IV - -(a) Give an immediate unsynchronized high-energy shock (defibrillation dose) A patient has sinus bradycardia w/ a heart rate of 36. Atropine has been administered to a total dose of 3 mg. A transcutaneous pacemaker has failed to capture. The pt is confused, and her BP is 88/56. Which therapy is now indicated? (a) Atropine 1 mg (b) Epinephrine 2 to 10 mcg/min (c) Adenosine 6 mg (d) Normal saline 250 mL to 500 mL bolus - -(b) Epinephrine 2 to 10 mcg/min A patient has sinus bradycardia with a heart rate of 36/min. Atropine has been administered to a toal does of 3 mg. A transcutaneous pacemaker has failed to capture. The patient is confused, and her blood pressure is 88/56 mm Hg. Which therapy is now indicated? - -Epinephrine 2 to 10 mcg/min A patient has sinus bradycardia with a heart rate of 36/min. Atropine has been administered to a total dose of 3 mg. A transcutaneous pacemaker has failed to capture. The patient is confused, and her blood pressure is 100/60 mm HG. What is now indicated? - -EPI 2-10 mcg/min A patient has sinus bradycardia with a heart rate of 36/min. Atropine has been administered to a total dose of 3 mg. A transcutaneous pacemaker has failed to capture. The patient is confused, and her blood pressure is 100/60 mm HG. What is now indicated? - -Epinephrine 2 to 10 mcg/min A patient has sudden onset of dizziness. The patient's heart rate is 180/min, blood pressure is 110/70 mm Hg, respiratory rate is 18 breaths/min, and pulse oximetry reading is 98% on room air. The lead II ECG is shown below: - -Vagal manuever. A patient in respiratory distress and with a blood pressure of 70/50 mm Hg presents with the following lead II ECG rhythm: What is the appropriate next intervention? - -Synchronized cardioversion A patient in respiratory failure becomes apneic but continues to have a strong pulse. The heart rate is dropping rapidly and now shows a sinus bradycardia at a rate of 30/min. What intervention has the highest priority? - -Simple airway manuevers and assisted ventilations. A patient is in cardiac arrest. High quality chest compressions are being given. The patient is intubated and an IV has been started. The rhythm is asystole. Which is the first drug/dose to administer? - -Epinephrine 1 mg or vasopressin 40 units IV or IO A patient is in cardiac arrest. High-quality chest compressions are being given. The patient is intubated, and an IV has been started. The rhythm is asystole. What is the first drug/dose to administer? - -Epinephrine 1mg IV/IO A patient is in cardiac arrest. Ventricular fibrillation has been refractory to a second shock. Which drug and dose should be administered first by the IV/IO route? - -Epinephrine 1 mg A patient is in cardiac arrest. Ventricular fibrillation has been refractory to a second shock. Which drug should be administered first? (a) Atropine 1 mg IV/IO (b) Epinephrine 1 mg IV/IO (c) Lidocaine 1 mg/kg IV/IO (d) Sodium bicarbonate 50 mEq IV/IO - -(b) Epinephrine 1 mg IV/IO A patient is in cardiac arrest. Ventricular fibrillation has been refractory to a second shock. Which drug should be administered first? - -Epinephrine 1 mg IV/IO A patient is in cardiac arrest. Ventricular fibrillation has been refractory to an initial shock. If no pathway for medication administration is in place, which method is preferred? - -IV or IO a patient is in cardiac arrest. ventricular fibrillation has been refractory to an initial shock. if no pathway for medication is in place, preferred method? - -IV or IO A patient is in cardiac arrest. Ventricular fibrillation has been refractory to an initial shock. What is the recommended route for drug administration during CPR? - -IV or IO A patient is in pulseless ventricular tachycardia. Two shocks and 1 dose of epinephrine has been given. Which is the next drug to anticipate to administer? - -amiodarone 300 mg A patient is in pulseless ventricular tachycardia. Two shocks and 1 dose of epinephrine have been given. Which drug should be given next? (a) Adenosine 6 mg (b) Amiodarone 300 mg (c) Epinephrine 3 mg (d) Lidocaine 0.5 mg/kg - -(b) Amiodarone 300 mg A patient is in pulseless ventricular tachycardia. Two shocks and 1 dose of epinephrine have been given. Which drug should be given next? - -Amiodarone 300mg A patient is in pulseless ventricular tachycardia. Two shocks and 1 dose of epinephrine have been given. Which is the next drug/dose to anticipate to administer? - -Amiodarone 300 mg A patient is in refractory ventricular fibrillation and has received multiple appropriate defibrillation shocks, epinephrine 1 mg IV twice, and an initial dose of 300 mg amiodarone IV. The patient is intubated A second dose of amiodarone is now called for. The recommended second dose of amiodarone is what? - -150 mg IV push A patient is in refractory ventricular fibrillation and has received multiple appropriate defibrillation shocks, epinephrine 1 mg IV twice, and an initial dose of 300 mg amiodarone IV. The patient is intubated. A second dose of amiodarone is now called for. The recommend second dose of amiodarone is ? - -150 mg IV push A patient is in refractory ventricular fibrillation and has received multiple appropriate defribillation shocks, epinephrine 1 mg IV twice, and an initial dose of amiodarone 300mg IV. The patient is intubated. Which best describe the recommended second does of amiodarone for this patient? - -150 mg IV push A patient is in refractory ventricular fibrillation. High CPR is in progress and shocks have been given. One dose of epinephrine was given after the second shock. An anti arrhythmic drug was given immediately after the the third shock. What drug should the team leader request to be prepared for administration next? - -second dose of epinephrine 1 mg A patient is in refractory ventricular fibrillation. High quality CPR is in progress, and shocks have been given. One dose of epinephrine was given after the second shock. An antiarrhythmic drug was given immediately after the third shock. What drug should the team leader request to be prepared for administration next? - -Second dose of epinephrine 1 mg A patient is in refractory ventricular fibrillation. High-quality CPR is in progress. One does of epinephrine was given after the second shock. An antiarrhythmic drug was given immediately after the third shock. You are the team leader. Which medication do you order next. - -Epinephrine 1 mg A patient presents to the emergency department with new onset of dizziness and fatigue. On examination, the patient's heart rate is 35/min, the blood pressure is 70/50 mm Hg, the respiratory rate is 22 breaths/min, and the oxygen saturation is 95%. What is the appropriate first medication? - -Atropine 0.5mg A patient remains in ventricular fibrillation despite 1 shock and 2 minutes of continuous CPR. The next intervention is to - -administer a second shock. A patient was in refractory ventricular fibrillation. A third shock has just been administered. Your team looks to you for instructions. Your immediate next order is? - -resume high quality chest compressions A patient was in refractory ventricular fibrillation. A third shock has just been administered. Your team looks to you for instructions. What is your next action? (a) Check the carotid pulse (b) Give amiodarone 300 mg IV (c) Give atropine 1 mg IV (d) Resume high-quality chest compressions - -(d) Resume high-quality chest compressions A patient was in refractory ventricular fibrillation. A thrid shock has just been administered. Your team looks to you for instructions. What is your next action? - -Resume high quality chest compressions A patient with a possible acute coronary syndrome has ongoing chest discomfort unresponsive to 3 SL NTG tablets. There are no contraindications, and 4 mg of morphine sulfate

Show more Read less
Institution
ACLS
Course
ACLS











Whoops! We can’t load your doc right now. Try again or contact support.

Document information

Uploaded on
February 16, 2023
Number of pages
40
Written in
2022/2023
Type
Exam (elaborations)
Contains
Questions & answers

Subjects

Content preview

ACLS COMPLETE latest Top Exam
Questions and answers, 100% Accurate.
Approved.

__________ can help indicate coronary perfusion pressure - ✔✔-Capnography



___________ correlates w/ ROSC - ✔✔-High quality CPR



2nd degree mobitz - ✔✔-



2nd degree wenckebach - ✔✔-



62 yr old man suddenly expereinced difficulty speaking and left-sided weakness. He meets initial criteria
for fibrinolytic therapy, and a CT scan of the brain is ordered. Which best describes the guidelines for
antiplatelet and fibrinolytic therapy?

(a) Give aspirin 160 to 325 mg to be chewed immediately

(b) Give aspirin 160 mg and clopidogrel 75 mg orally

(c) Give heparin if the CT scan is negative for hemorrhage

(d) Hold aspirin for at least 24 hrs if rtPA is administered - ✔✔-(d) Hold aspirin for at least 24 hrs if rtPA
is administered



A 35 year old woman has palpitations, light headedness, and a stable tachycardia. The monitor shows a
regular narrow-complex QRS at a rate of 180/min. Vagal maneuvers have not been effective in
terminating the rhythm. An IV has been established. What drug should be administered IV? - ✔✔-
Adenosine 6 mg



A 35 year old woman has palpitations, light headedness, and a stable tachycardia. The monitor shows a
regular narrow-complex QRS at a rate of 180/min. Vagal maneuvers have not been effective in
terminating the rhythm. An IV has been established. What drug should be administered? - ✔✔-
adenosine 6 mg

,A 35 yr old female has palpitation, light-headedness, and a stable tachycardia. The monitor shows a
regular narrow-complex QRS at a rate of 180/min. Vagal manuevers have not been effective in
terminating the rhythm. An IV has been established. Which drug should be administered?

(a) Adenosine 6 mg

(b) Atropine 0.5 mg

(c) Epinephrine 2 to 10 mcg/kg per minute

(d) Lidocaine - ✔✔-(a) Adenosine 6 mg



A 35 yr old woman presents w/ a chief complaint of palpitations. She has no chest discomfort, shortness
of breath, or light-headedness. Her BP is 120/78. On EKG, it shows she is in SVT. Which intervention is
indicated first?

(a) Adenosine 3 mg IV bolus

(b) Adenosine 12 mg IV slow push (over 1 to 2 min)

(c) Metoprolol 5 mg IV and repeat if necessary

(d) Vagal manuevers - ✔✔-(d) Vagal manuevers



A 35-year-old woman has palpitations, light-headedness, and a stable tachycardia. The monitor shows a
regular narrow-complex QRS at a rate of 180/min. Vagal maneuvers have not been effective in
terminating the rhythm. An IV has been established. Which drug should be administered? - ✔✔-
Adenosine 6mg



A 35-year-old woman presents with a chief complaint of palpitations. She has no chest discomfort,
shortness of breath, or light-headedness. Her blood pressure is 120/78mm Hg. Which intervention is
indicated first? - ✔✔-Vagal maneuvers



a 45 year old woman has a history of palpitations develops lightheadedness and palpitations. She has
received adenosine 6 mg IV for SVT without conversion. BP 128/70. next step? - ✔✔-adenosine 12 mg



A 45 yr old woman with a hx of palpitations develops light-headedness and palpitations. She has
received adenosine 6 mg IV for the rhythm shown here, without conversion of the rhythm. She is now
extremely apprehensive. Her BP is 128/70 mm Hg. What is the next appropriate intervention?

(a) Administer adenosine 12 mg IV

,(b) Perform unsynchronized cardioversion

(c) Perform vagal maneuvers

(d) Perform synchronized cardioversion - ✔✔-(a) Administer adenosine 12 mg IV



A 45-year-old woman with a history of palpitations develops light-headedness and palpitations. She has
received adensoine 6mg IV for the rhythm shown here (SVT), without conversion of the rhythm. She is
now extremely apprehensive. Her blood pressure si 128/70mm Hg. What is the next appropriate
intervention? - ✔✔-Administer adenosine 12 mg IV



A 49-year-old man has retrosternal chest pain radiating into the left arm. The patient is diaphoretic, with
associated shortness of breath. The blood pressure is 130/88 mm Hg, the heart rate is 110/min, the
respiratory rate is 22 breaths/min, and the pulse oximetry value is 95%. The patient's 12-lead ECG shows
ST-segment elevation in the anterior leads. First responders administered 160 mg of aspirin, and there is
a patent peripheral IV. The pain is described as an 8 on a scale of 1 to 10 and is unrelieved after 3 doses
of nitroglycerin. What is the next action? - ✔✔-Administer 2 to 4 mg of morphine by slow IV bolus.



A 49-year-old woman arrives in the emergency department with persistent epigastric pain. She had
been taking oral antacids for the past 6 hours because she thought she had heartburn. The initial blood
pressure is 118/72 mm Hg, the heart rate is 92/min and regular, the nonlabored respiratory rate is 14
breaths/min, and the pulse oximetry reading is 96%. Which is the most appropriate intervention to
perform next? - ✔✔-Obtain a 12 lead ECG.



A 53-year-old man has shortness of breath, chest discomfort, and weakness. The patient's blood
pressure is 102/59 mm Hg, the heart rate is 230/min, the respiratory rate is 16 breaths/min, and the
pulse oximetry reading is 96%. The lead II ECG is displayed below. A patent peripheral IV is in place.
What is the next action? - ✔✔-Vagal maneuvers



A 56-year-old man reports that he has palpitations but not chest pain or difficulty breathing. The blood
pressure is 132/68 mm Hg, the pulse is 130/min and regular, the respiratory rate is 12 breaths/min, and
the pulse oximetry reading is 95%. The lead II ECG displays a wide-complex tachycardia. What is the next
action after establishing an IV and obtaining a 12-lead ECG? - ✔✔-Seeking expert consultation



A 57 year old woman has palpitations, chest discomfort, and tachycardia. The monitor shows a regular
wide-complex QRS at a rate of 180 bpm. She becomes diaphoretic and her blood pressure is 80/60 mm
Hg. What is the next action? - ✔✔-cardioversion

, A 57 year old woman has palpitations, chest discomfort, and tachycardia. The monitor shows a regular
wide-complex QRS at a rate of 180 bpm. She becomes diaphoretic and her blood pressure is 80/60 mm
Hg. What is the next action? - ✔✔-Perform immediate electrical cardioversion



A 57-year-old woman has palpitation, chest discomfort, and tachycardia. The monitor shows a regular
wide-QRS at a rate of 180/min. She becomes diaphoretic, and her blood pressure is 80/60 mm HG/
Which action do you take next? - ✔✔-Perform electrical cordioversion



A 62 year old man suddenly experienced difficulty speaking and left-sided weakness. He was brought
into the emergency department. He meets initial criteria for fibrinolytic therapy, and a CT scan of the
brain is ordered. What are the guidelines for antiplatelet and fibrinolytic therapy? - ✔✔-Do not give ASA
for at least 24 hours if rtPA is administered



A 62 year old man suddenly experienced difficulty speaking and left-sided weakness. He was brought
into the emergency department. He meets initial criteria for fibrinolytic therapy, and a CT scan of the
brain is ordered. What are the guidelines for antiplatelet and fibrinolytic therapy? - ✔✔-hold aspirin for
at least 24 hours if rtPA is administered



A 62-year-old man in the emergency department says that his heart is beating fast. He says he has no
chest pain or shortness of breath. The blood pressure is 142/98 mm Hg, the pulse is 200/min, the
respiratory rate is 14 breaths/min, and pulse oximetry is 95% on room air. What intervention should you
perform next? - ✔✔-Obtain a 12 lead ECG.



A 62-year-old man suddenly experienced difficulty speaking and left-sided weakness. He meets initial
criteria for fibrinolytic therapy, and a CT scan of the brain si ordered. Which best describes the
guidelines for antiplatelet and fibrinolytic therapy? - ✔✔-Hold aspirin for at least 24 hours if rtPA is
administered



A 67 yr old woman has palpitations, chest discomfort, and tachycardia. The monitor shows a regular
wide-complex QRS at a rate of 180/min. She becomes diaphoretic, and her blood pressure is 80/60.
Which action do you take next?

(a) Establish IV access

(b) Obtain a 12 lead EKG

(c) Perform electrical cardioversion

Get to know the seller

Seller avatar
Reputation scores are based on the amount of documents a seller has sold for a fee and the reviews they have received for those documents. There are three levels: Bronze, Silver and Gold. The better the reputation, the more your can rely on the quality of the sellers work.
QuickPass Chamberlain College Of Nursing
View profile
Follow You need to be logged in order to follow users or courses
Sold
410
Member since
3 year
Number of followers
304
Documents
6372
Last sold
2 weeks ago
Get all Documents you need at discount.

All nursing docs available: question banks, summaries, study guide and test banks

3.8

97 reviews

5
52
4
11
3
10
2
10
1
14

Recently viewed by you

Why students choose Stuvia

Created by fellow students, verified by reviews

Quality you can trust: written by students who passed their tests and reviewed by others who've used these notes.

Didn't get what you expected? Choose another document

No worries! You can instantly pick a different document that better fits what you're looking for.

Pay as you like, start learning right away

No subscription, no commitments. Pay the way you're used to via credit card and download your PDF document instantly.

Student with book image

“Bought, downloaded, and aced it. It really can be that simple.”

Alisha Student

Frequently asked questions